The Collectors

Bài 5 trang 68 SGK Hình học 12

Câu hỏi: Tìm tâm và bán kính của các mặt cầu có phương trình sau đây:

Câu a​

a) \({x^2} + {\rm{ }}{y^{2}} + {\rm{ }}{z^2}-{\rm{ }}8x{\rm{ }} - {\rm{ }}2y{\rm{ }} + {\rm{ }}1{\rm{ }} = {\rm{ }}0\);
Phương pháp giải:
Cách 1: Đưa phương trình về dạng phương trình chính tắc: \({\left( {x - a} \right)^2} + {\left({y - b} \right)^2} + {\left({z - c} \right)^2} = {R^2}\), suy ra tâm \(I\left( {a; b; c} \right)\) và bán kính bằng \(R\).
Cách 2: Phương trình có dạng \({x^2} + {y^2} + {z^2} + 2ax + 2by + 2cz + d = 0 \left( {{a^2} + {b^2} + {c^2} - d > 0} \right)\) là phương trình mặt cầu có tâm \(I\left( {-a;-b;-c} \right)\) và bán kính \(R = \sqrt {{a^2} + {b^2} + {c^2} - d} \).
Lời giải chi tiết:
Cách 1: Ta có phương trình : \({x^2} + {\rm{ }}{y^{2}} + {\rm{ }}{z^2}-{\rm{ }}8x{\rm{ }} - {\rm{ }}2y{\rm{ }} + {\rm{ }}1{\rm{ }} = {\rm{ }}0\)
\(\Leftrightarrow {\rm{ }}{\left( {x{\rm{ }}-{\rm{ }}4} \right)^2} + {\rm{ }}{\left({y{\rm{ }}-{\rm{ }}1} \right)^2} + {\rm{ }}{z^2} = {\rm{ }}{4^2}\)
Đây là mặt cầu tâm \(I(4; 1; 0)\) và có bán kính \(r = 4\).
Cách 2: Ta có: \(a = -4; b = -1; c = 0  ; d = 1 \Rightarrow {a^2} + {b^2} + {c^2} - d = 16 > 0\) do đó đây là phương trình mặt cầu tâm \(I\left( {4; 1; 0} \right)\), bán kính \(R=4\).

Câu b​

b) \(3{x^2} + {\rm{ }}3{y^2} + {\rm{ }}3{z^2}-{\rm{ }}6x{\rm{ }} + {\rm{ }}8y{\rm{ }} + {\rm{ }}15z{\rm{ }}-{\rm{ }}3{\rm{ }} = {\rm{ }}0\)
Lời giải chi tiết:
Cách 1: Ta có phương trình:
\(3{x^2} + {\rm{ }}3{y^2} + {\rm{ }}3{z^2}-{\rm{ }}6x{\rm{ }} + {\rm{ }}8y{\rm{ }} + {\rm{ }}15z{\rm{ }}-{\rm{ }}3{\rm{ }} = {\rm{ }}0\)
\(\Leftrightarrow {x^2} + {y^2} + {z^2}{\rm{  - }}2x + {8 \over 3}y + 5z{\rm{  - }}1 = 0\)
\(⇔ (x-1)^{2}+(y+\dfrac{4}{3})^{2}+(z+\dfrac{5}{2})^{2}= (\dfrac{19}{6})^{2}\).
Đây là mặt cầu tâm \(J(1; -\dfrac{4}{3};-\dfrac{5}{2})\) và có bán kính là \(R = \dfrac{19}{6}\).
Cách 2:
\(\begin{array}{l} 3{x^2} + 3{y^2} + 3{z^2} - 6x + 8y + 15z - 3 = 0\\\Leftrightarrow {x^2} + {y^2} + {z^2} - 2x + \dfrac{8}{3}y + 5z - 1 = 0
\end{array}\)
Ta có: \(a = -1; b =   \dfrac{4}{3}; c =  \dfrac{5}{2}; d =  - 1 \) \(\Rightarrow {a^2} + {b^2} + {c^2} - d = \dfrac{{361}}{{36}} > 0\) do đó đây là phương trình mặt cầu tâm \(J\left( {1; - \dfrac{4}{3}; - \dfrac{5}{2}} \right)\), bán kính \(R = \dfrac{{19}}{6}\).
Rất tiếc, câu hỏi này chưa có lời giải chi tiết. Bạn ơi, đăng nhập và giải chi tiết giúp zix.vn nhé!!!
 

Quảng cáo

Back
Top